kimnamil14
Thanks Received: 2
Forum Guests
 
Posts: 12
Joined: September 07th, 2010
 
 
 

Q17 - Ecologist: Forest fires, the vast

by kimnamil14 Sun Sep 12, 2010 2:59 am

I am having a hard time trying to decide between A and B on this one.

When I first looked at the question, I thought the ecologist is assuming that human intervention to stop or control forest fires has negative consequences for the environment. So A looked pretty tempting. But then I'm thinking, isn't this a necessary assumption?? So I looked at B, and B seems convincing even though it seems to imply that human intervention to stop or control forest fires is not for protection of forest and their ecosystems.

Can someone please clarify my confusion? Thank you so much!
 
aileenann
Thanks Received: 227
Atticus Finch
Atticus Finch
 
Posts: 300
Joined: March 10th, 2009
 
This post thanked 1 time.
 
 

Re: Q17 - Ecologist: Forest fires, the vast

by aileenann Mon Sep 13, 2010 3:49 pm

You are absolutely right that this question is asking for a sufficient assumption, not just a necessary one. Still, on my first pass through the questions, I'm just looking for anything that looks close- the sufficient v. necessary distinction shouldn't be super important at the outset. Rather, I will use this distinction when I am down to my last couple of answer choices after a first round of elimination.

First, a look at (A). I would say it's not the answer for a couple of reasons. One is that it isn't sufficient - it only covers biodiversity, for one thing, while the argument itself focuses on several benefits from forest fires, not just one.

Second, even if this is really specific and hence not sufficient, I also don't think this is a necessary assumption. If you negate it, it just tells us that human interference with natural properties doesn't tend to reduce diversity. But even if this is true as a tendency,what about the specific case of fires? And more than that, does this really get to what we care about - that is, some link between humans and whether they ought to be interfering. It seems to me we are looking for something a little more normative.

In contrast, (B), the correct answer, has got a lot more of what I am looking for. First, it's got the sort of normative language ("only legitimate reasons") that we should be looking for based on the language of the argument's conclusion. Second, it does all the heavy lifting this argument needs. That is, if the benefit to forests is the only reason to interfere - but we know fires do just fine benefiting forests without humans - so therefore there is no good reason to interfere. Voila, here is our conclusion.

Does that make sense? Please let me know if you have follow-up questions or comments!
 
slimjimsquinn
Thanks Received: 1
Forum Guests
 
Posts: 43
Joined: February 11th, 2012
 
 
 

Re: Q17 - Ecologist: Forest fires, the vast

by slimjimsquinn Wed Jul 18, 2012 11:54 pm

I eliminated B) because I didn't think the stimulus made it clear that "protection of forest" included facilitating the opening of pods, preventing overabundance of insects, etc. The first sentence also says that the list above are things required for a forest to FLOURISH.

I wasn't convinced that protection equaled flourish or the list above. Am I over thinking things?
 
giladedelman
Thanks Received: 833
LSAT Geek
 
Posts: 619
Joined: April 04th, 2010
 
 
 

Re: Q17 - Ecologist: Forest fires, the vast

by giladedelman Sun Jul 22, 2012 11:29 am

Yes, you are overthinking things. It's not that forest fires themselves "protect" forest ecosystems. It's that, since forest fires have a bunch of positive effects on forests that help forests survive, we know that trying to prevent and control forest fires will definitely NOT tend to protect the forests--in fact, it will probably have a negative effect. So if we assume that protecting the forests is the ONLY legitimate reason to control forest fires, then we can conclude that we shouldn't be trying to do that, since it will not lead to the protection of forests.

Does that answer your question?
 
gplaya123
Thanks Received: 15
Forum Guests
 
Posts: 90
Joined: September 04th, 2012
 
 
 

Re: Q17 - Ecologist: Forest fires, the vast

by gplaya123 Tue Mar 19, 2013 1:19 am

Please help! I really don't understand how B is the actual answer... the below is the only way that I make myself believe that B is the answer (by using the formal logic)

let me diagram answer choice B: if forest ever needs a prevention -> then it’s to protect the forest (or more like it’s to benefit the forest).
Now, the original argument is this: if the fire benefit the forest -> then the forest does not need a prevention.
So if B is established, and combined with the argument, it says this:
If forest ever needs a prevention -> then it’s to benefit the forest -> fire benefits the forest -> therefore it does not need prevention.
Is my reasoning correct?
 
timsportschuetz
Thanks Received: 46
Elle Woods
Elle Woods
 
Posts: 95
Joined: June 30th, 2013
 
 
trophy
First Responder
 

Re: Q17 - Ecologist: Forest fires, the vast

by timsportschuetz Fri Nov 08, 2013 11:51 pm

Can someone from the Manhattan team PLEASE explain this question to me!!! I eliminated every answer and eventually chose E since it is the only one mentioning the conclusion that humans should leave forest fires alone.
 
christine.defenbaugh
Thanks Received: 585
Atticus Finch
Atticus Finch
 
Posts: 536
Joined: May 17th, 2013
 
This post thanked 3 times.
 
 

Re: Q17 - Ecologist: Forest fires, the vast

by christine.defenbaugh Mon Nov 11, 2013 1:15 pm

timsportschuetz Wrote:Can someone from the Manhattan team PLEASE explain this question to me!!! I eliminated every answer and eventually chose E since it is the only one mentioning the conclusion that humans should leave forest fires alone.


This is a tricky sufficient assumption question!

The key here is interpreting the core. There's a ton of information, but most of it can get simplified down to a very basic idea:

    Premise: Forest fires benefit forests and their ecosystems (and are required for those to flourish).
    Conclusion: We shouldn't control/prevent forest fires.
Note that all the specifics in the middle of the stimulus are really just examples of how forest fires benefit forests.

So what we have here is one good characteristic of a thing, and then a conclusion that we should never control or prevent that thing. That's quite a leap! Aren't there a ton of other reasons that might justify preventing forest fires? Like, protecting homes nearby, or campers?

But......if we add in that protecting forests is the only possible legitimate reason for controlling/preventing forest fires, we dismiss all those other potential reasons in one fell swoop. So, there's only one legit reason to control/prevent fires, and that's if it protects the forest. But our premise said fires are GOOD for forests! So that leaves exactly zero legit reasons to control/prevent forest fires. We've arrived at our conclusion! (B) is our Sufficient Assumption!





I understand the desire to look for an answer choice that has the classic "If (premise), then (conclusion)" structure to it, but while that's a common theme it's not a requirement. (E) gets to the correct result/conclusion, but the trigger/premise is completely wrong.

It's important to use this kind of pattern recognition to help guide you to common forms of correct answers, but be careful not to be so wed to it that you inadvertantly focus on the form over content!

Please let me know if this completely answers your question!
User avatar
 
Mab6q
Thanks Received: 31
Atticus Finch
Atticus Finch
 
Posts: 290
Joined: June 30th, 2013
 
 
 

Re: Q17 - Ecologist: Forest fires, the vast

by Mab6q Thu Jan 22, 2015 12:34 am

This question can very easy be answered by focusing on the conditional in B:

legitimate reason for attempting to control or prevent fires --> protection of forests and their ecosystems.

Now, from the stimulus we are told that efforts taken to control forest fires are ill-advised. Although some may take this to be part of the conclusion, I consider it to be more of a premise so we take it to be true.

SO, if human action is ill-advised, we know it is not protecting forests and their ecosystem (the necessary conclusion). That triggers our contrapositive, which gives us ~legitimate reason for controlling. That is very close to our conclusion that they should be left alone.

Cheers.
"Just keep swimming"
 
SJK493
Thanks Received: 1
Jackie Chiles
Jackie Chiles
 
Posts: 31
Joined: May 14th, 2018
 
 
 

Re: Q17 - Ecologist: Forest fires, the vast

by SJK493 Thu Aug 16, 2018 1:19 am

Is there an explanation to eliminate all the other answers?
Would appreciate any help! Thanks!